Đến nội dung

no matter what

no matter what

Đăng ký: 07-07-2012
Offline Đăng nhập: 25-06-2015 - 21:13
****-

#383010 $\sqrt{ab(a+b)}+\sqrt{bc(b+c)}+\sqrt...

Gửi bởi no matter what trong 02-01-2013 - 20:42

Cho $a,b,c>0$.CMR
$\sqrt{ab(a+b)}+\sqrt{bc(b+c)}+\sqrt{ca(c+a)}\geq \sqrt{4abc+(a+b)(b+c)(c+a)}$

+,Xét với $a=b=c=0$,BĐT đúng
+,Với a,b,c khác 0
Bình phương 2 vế và chú ý đẳng thức $\sum ab(a+b)=\prod (a+b)-2abc$
ta chỉ cần chứng minh BĐT $\sqrt{abc}(\sqrt{b(a+b)(b+c)}+\sqrt{c(b+c)(c+a)}+\sqrt{a(a+b)(c+a)}\geq 3abc$
hay $\sqrt{b(a+b)(b+c)}+\sqrt{c(b+c)(c+a)}+\sqrt{a(a+b)(c+a)}\geq 3\sqrt{abc}$
Theo AM-GM thì BĐT trên đúng (nhưng không xảy ra đẳng thức )


#382854 Mỗi tuần một ca khúc!

Gửi bởi no matter what trong 02-01-2013 - 13:01

We are young nhé :luoi:



#382849 $\frac{1}{a}+\frac{1}{b...

Gửi bởi no matter what trong 02-01-2013 - 12:45

Cho $a,b,c>0$. CMR
$\frac{1}{a}+\frac{1}{b}+\frac{1}{c}\geq \frac{a+b}{c^{2} +ab}+\frac{b+c}{a^{2}+bc}+\frac{c+a}{b^{2}+ca}$

Nếu vết $\frac{1}{c}-\frac{a+b}{c^2+ab}$$=\frac{(c-b)(c-a)}{c^2+ab}$ thì BĐT hiển nhiên đúng theo schur suy rộng :icon6:


#382041 Chứng minh rằng : $\sum \frac{a^2}{2a^2+(b+c)^2...

Gửi bởi no matter what trong 30-12-2012 - 18:44

Cho $a$ $,$ $b$ $,$ $c$ là các số thực không âm. Chứng minh rằng :
$\frac{a^2}{2a^2+(b+c)^2}+\frac{b^2}{2b^2+(a+c)^2}+\frac{c^2}{2c^2+(b+a)^2} \leq \frac{2}{3}$.

http://diendantoanho...2bc2leq-frac12/
thiếu dk đánh giá bạn

Cho mình biết khi nào dấu bằng xảy ra đc ko bạn? mình nghĩ là $\leq \frac{1}{2}$ chứ nhỉ?

có phải khi nào đẳng thức cũng xảy ra khi các biến bằng nhau đâu?


#381552 Mỗi tuần một ca khúc!

Gửi bởi no matter what trong 29-12-2012 - 12:49

Winter in my Heart

Không ngờ vẫn còn người thích thể loại nhạc này :icon6: (ngoài mình :nav: )
SORRY THAT I LOVED YOU

  • MIM yêu thích


#381276 BĐT AM-GM

Gửi bởi no matter what trong 28-12-2012 - 19:20

Bài 11,,Chứng mnih với mọi a,b,c thực dương
$\sqrt{\frac{a^2+2b^2}{a^2+ab+bc}}+\sqrt{\frac{b^2+2c^2}{b^2+bc+ca}}+\sqrt{\frac{c^2+2a^2}{c^2+ca+ab}}\geq 3$
Gợi ý:Có thể tham khảo cách giải của duong vi tuan trong các bài 12,13 trang 3 :icon6:


#381000 BĐT AM-GM

Gửi bởi no matter what trong 27-12-2012 - 20:47

Sao cái này tôi đặt mà nó lại không biến đổi ra được thế này.lạ nhỉ?? :wacko:

mình cũng chả biến đổi ra nữa :icon6: (trích sáng tạo BĐT)
Bài 10,Chứng minh với mọi x,y,z $> -1$,ta có
$\frac{1+x^2}{1+y+z^2}+\frac{1+y^2}{1+z+x^2}+\frac{1+z^2}{1+x+y^2}\geq 2$
Mình nghĩ bài này không cần gợi ý,chỉ tại mình người chưa đọc đến thôi,chỉ cần AM-GM đưa về đồng bậc là xem như ổn định :icon6:


#380737 BĐT AM-GM

Gửi bởi no matter what trong 26-12-2012 - 22:16

:icon6: giải quyết nhanh gọn
Bài 5,Với mọi x,y,z có $x+2y+3z=\frac{1}{4}$,tìm MAX
$\frac{232y^3-x^3}{2xy+24y^2}+\frac{783z^3-8y^3}{6yz+54z^2}+\frac{29x^3-27z^3}{3xz+6x^2}$
Gợi ý:,ta chỉ cần đặt $x=a,2y=b,3z=c$,mọi người làm tiếp nhé :wub:
Sau khi đổi biến,ta sẽ thu đương BĐT "tương tự " như thế này
http://diendantoanho...5b2-leq-3sum-a/


#380724 BĐT AM-GM

Gửi bởi no matter what trong 26-12-2012 - 21:53

xoắn quá,lại 1 hình thức tự xướng rồi
Giải bài 1(lơig giải trong sáng tạo BĐT
:đk của đề bài đẫn đến việc tồn tại x,y,z sao cho$a=\frac{x}{y},b=\frac{y}{z},c=\frac{z}{x}$
BĐT trở thành $x^3+y^3+z^3+3xyz\geq xy(x+y)+yz(y+z)+zx(z+x)$ (Schur bậc 3)
Ngoài ra,ta có thể chú ý đến biến đổi đơn giản sau
$\frac{a+b}{c}+\frac{b+c}{a}+\frac{c+a}{b}+6=(a+b+c)(\frac{1}{a}+\frac{1}{b}+\frac{1}{c})+3$
Đến đây chỉ cần chuyển biến của BĐT về $p,q,r$(đã nêu ở trang 1) với chú ý r=1,BĐT còn lại chứng minh không hề khó khăn
Bài 3 :,Chứng minh với mọi a,b,c có $0< a,b,c\leq \frac{1}{3},a^3+b^3+c^3=\frac{3}{64}$ thì
$\frac{1}{1-3a}+\frac{1}{1-3b}+\frac{1}{1-3c}\geq 12$
Gợi ý :Ta chứng minh BĐT phụ sau $a^3(1-3a)\leq \frac{1}{256}$
p/s:Mong rằng mnihf sẽ không phải tự sướng tiếp :(


#380652 $\sum \sqrt{\frac{a+b}{2c}}...

Gửi bởi no matter what trong 26-12-2012 - 19:12

Chứng minh với mọi a,b,c dương có $a^3+b^3+c^3+3c=5$,ta có BĐT sau
$\sqrt{\frac{a+b}{2c}}+\sqrt{\frac{b+c}{2a}}+\sqrt{\frac{c+a}{2b}}< \frac{1}{a}+\frac{1}{b}+\frac{1}{c}$



#380627 $\prod \left ( a + b \right ) \leq \prod \...

Gửi bởi no matter what trong 26-12-2012 - 17:09

Cho a,b,c>0. a+b+c=3. Chứng minh bất đẳng thức:
$(a+b)(b+c)(c+a)\geq (a+bc)(b+ac)(c+ab)$
Bài này mình nghĩ dùng kĩ thuật ghép đối xứng nhưng quả thật là mình không biết ghép thế nào

Giaỉ
Áp dụng AM-GM $(a+bc)(b+ca)\leq (\frac{a+b+c(a+b)}{2})^2=\frac{(a+b)^2(c+1)^2}{4}$
Tương tự, ta chỉ cần chứng minh $(a+b)(b+c)(c+a)\geq \frac{(a+b)(b+c)(c+a)(a+1)(b+1)(c+1)}{8}$
hay $(a+1((b+1)(c+1)\leq 8$
Tuy nhiên BĐT này hiển nhiên đúng với a,b,c dương có tổng bằng 3


#380473 tìm min $\frac{x}{{\sqrt y }} +...

Gửi bởi no matter what trong 25-12-2012 - 21:52

Kí hiệu $\sum$ cho nhanh nhé :icon6:
Thực chất chỉ là bình phương +AM-GM:
Với P là bt đã cho,có $P^2=\sum \frac{x^2}{y}+\sum \frac{2x\sqrt{y}}{\sqrt{z}}$
Có tiếp $\frac{x^2}{y}+\frac{2x\sqrt{y}}{\sqrt{z}}+z=\frac{x^2}{y}+\frac{x\sqrt{y}}{\sqrt{z}}+\frac{x\sqrt{y}}{\sqrt{z}}+z\geq 4\sqrt[4]{...}=4x$
Xây dựng các BĐT tương tự kết hợp với dk là có $P\geq 6$
P/s:Hình như còn các C-S


#380389 BĐT AM-GM

Gửi bởi no matter what trong 25-12-2012 - 20:03

Trong điều kiện có thể,mình nghĩ ta nên dọn dẹp topic 1 lượt đã :lol:
Trước tiên là bài 1 -mình xn post lại đề ở đây

1,chứng minh với mọi số thực dương a,b,c có tích bằng 1 thì

$\frac{a+b}{c}+\frac{b+c}{a}+\frac{c+a}{b}+6\geq 2(a+b+c+\frac{1}{a}+\frac{1}{b}+\frac{1}{c})$
Gợi ý: Tư ttưởng của bài này thực chất đã khá rõ ràng khi cho tích bằng 1 ,các bạn nên liên tưởng tới kĩ thuật đặt ẩn ở phần đánh giá mẫu mnihf đã post ở trang 1
Mọi người hoàn thiện lời giải nhanh để ta chuyển bài nhé :lol:
Trong thời gian làm bài,nếu thất đau trốc thì cứ zào đây mà đọc,đảm bảo đỡ ngay
http://traitimnghiah...-binladen.2113/


#380385 BĐT AM-GM

Gửi bởi no matter what trong 25-12-2012 - 19:55

Lấy ý tưởng của no matter what
$ax^{n}+\frac{1}{bx^{k}}=k.\frac{a}{k}x^n+n.\frac{1}{nbx^k}\ge (n+k)\sqrt[n+k]{\frac{a^k}{k^k}.\frac{1}{n^n.b^k}}$
Còn vụ dấu bằng thì :(

Vụ dấu bằng để bác khác làm zậy :icon6:
làm cái này mnihf đau cả trốc ~O)


#380378 BĐT AM-GM

Gửi bởi no matter what trong 25-12-2012 - 19:49

*Để đánh dấu sự trơ lại sau những ngày âm u thất sắc và mọi người chỉ nằm co ro trong chăn mà không thèm dí tay vào cái bàn phím để đánh lời giải mấy bài trên cho mình,mình xin được đưa ra một bài sau:Khá nghiêm túc
Chứng minh với mọi a,b,c dương có $a^3+b^3+c^3+3c=5$,ta có BĐT sau
$\sqrt{\frac{a+b}{2c}}+\sqrt{\frac{b+c}{2a}}+\sqrt{\frac{c+a}{2b}}\leq \frac{1}{a}+\frac{1}{b}+\frac{1}{c}$

thực tế sau khi xem lời giải,các bạn sẽ thấy nó không khó,tuy nhiên để đến lời giải đó cũng không dễ :lol:
Mình sẽ xin đi lại hướng cũ của topic và không rong hoa bắt bướm nữa,những bài tập sau này(nếu do mình post)sẽ liên quan tới phần kiến thức đẫ nếu,nếu cần mnihf cũng sẽ nêu rõ phạm vi kiến thức :lol:
Mình sẽ cố gắng post lên về pp trọng só(hoặc nhờ người khác post)
Xin chân thành cảm ơn :nav: